Hipótesis de Riemann: ¿es el hamiltoniano de Bender-Brody-Müller una nueva línea de ataque?

Hay un hermoso artículo en Physical Review Letters [PRL 118, 130201 (2017), DOI:10.1103/PhysRevLett.118.130201] de Carl Bender, Dorje Brody y Markus Müller (BBM) sobre un enfoque hamiltoniano de la hipótesis de Riemann. El documento es sorprendentemente fácil de seguir para un físico.

BBM define un hamiltoniano

H ^ = ( 1 mi i pag ^ ) 1 ( X ^ pag ^ + pag ^ X ^ ) ( 1 mi i pag ^ )
dónde pag = i X es el operador de cantidad de movimiento en = 1 unidades.

Los autores muestran que las funciones propias de H ^ desaparecer en el infinito debe estar en la forma de la función theta de Hurwitz, ψ z = ζ ( z , X + 1 ) , de modo que

H ^ ψ z = i ( 2 z 1 ) ψ z

Imposición de una condición de contorno ψ z ( 0 ) = 0 , en virtud de ζ ( z , 1 ) = ζ ( z ) , muestran que todos los ceros no triviales del Riemann ζ función debe ser valores propios de H ^ con las condiciones de contorno impuestas.

BBM llama a este resultado "una versión extendida compleja de la conjetura de Berry-Keating" y continúa proporcionando argumentos heurísticos de que todos los valores propios de H ^ Son reales.

¿Qué tan prometedor es este nuevo desarrollo en el contexto de resolver la Hipótesis de Riemann ?

Steven Strogatz parece optimista .

Actualización [19.10.2020]: Los autores de [ 1 ] han publicado comentarios adicionales [ 3 , 4 ], incluida una respuesta [ 4 ] al comentario de @Jean Bellissard [ 5 ] que surgió de su respuesta a continuación.

"Optimista" lo está poniendo un poco fuerte. Es simplemente que siempre me ha intrigado esta línea de ataque desde que escuché a Michael Berry hablar sobre ella. Pero dejaré en manos de los expertos la cuestión de si es nuevo o realmente prometedor. ¿Alguien quiere pesarse? steven strogatz
Hay un comentario de Jean Bellissard en la publicación de FB de Sabine Hossenfelder (que espero que esté bien citar aquí): "Me parece que la hipótesis principal, que el operador de momento es autoadjunto, no se cumple. De hecho, la condición de frontera de Dirichlet en cero, dale un conjunto no trivial de índices de defectos que, por el teorema de Von Neumann, impiden que tenga una extensión autoadjunta". ¿Alguien puede convertir esto en una respuesta?
Aquí hay potencialmente algunos antecedentes interesantes y un intento de usar las funciones estándar de Hurwitz zeta: motls.blogspot.nl/2017/03/…
¿Puedes mostrar por qué? ψ ( X ) = ζ ( z , X + 1 ) debe ser una función propia de este operador diferencial? No estoy convencido de los ceros reales de Ξ ( t ) son los valores propios de un operador diferencial tan simple. El punto de partida parece ser [ Δ ^ F ] ( X ) = F ( X ) F ( X 1 ) , Δ ^ 1 X z = Γ ( 1 z ) 2 i π C t z 1 mi X t 1 mi t d t = Γ ( 1 z ) 2 i π Γ ( z ) ζ ( z , X ) dónde Δ ^ 1 es el operador inverso al observar las funciones analíticas que se desvanecen en +
@user1952009 Hasta el último paso, esto es exactamente lo que hace BBM. Según en.wikipedia.org/wiki/Hurwitz_zeta_function , Γ ( 1 z ) 2 i π C t z 1 mi X t 1 mi t d t = ζ ( z , X ) para z < 1 , aunque no veo por qué esto debería ser igual a ζ ( z , X + 1 ) .
Usa eso mi X t 1 mi t = norte = 0 mi ( norte X ) t y 0 t z 1 mi ( norte X ) t d t = ( norte X ) z Γ ( z ) .
Para una historia similar pero más antigua, consulte la sección 0.3 de las notas de Paul Garrett sobre formas de cúspide en www-users.math.umn.edu/~garrett/m/v/pseudo-cuspforms.pdf . Allí, como aquí, uno tiene los ceros zeta como valores propios provenientes de un problema espectral que, lamentablemente, no corresponde a un problema de valores propios autoadjuntos y, por lo tanto, no implica RH. En particular, señala específicamente que las funciones zeta de Epstein pueden fallar explícitamente en esta implicación debido a la falta de productos de Euler, análoga a la objeción planteada en la respuesta de @ user1952009.
Un poco de deja vu. Si la memoria no me falla, una vez vi un hamiltoniano que resuelve los números primos, hace unos veinte años.
Ahora también en discusión sobre MathOverflow, mathoverflow.net/questions/266935/…

Respuestas (10)

Traté de poner mi propio argumento en este blog recientemente, pero fracasé. Déjame intentar de nuevo. 1)- La definición original de ζ ( z , X ) es

ζ ( z , X ) = norte = 1 1 ( norte + X ) z

Entonces ζ ( z , X ) ζ ( z , X 1 ) = 1 / X z . Entonces, utilizando la representación integral, esta relación se extiende por analiticidad al dominio máximo en el que ambos lados están definidos y son holomorfos. Para nuestro propósito, es suficiente considerar X > 0 y z > 0 . Dejar ζ z denota el mapa X ζ ( z , X ) . Entonces, usando el generador A del operador de dilatación, es fácil comprobar que los valores propios tienen la forma 1 / 2 + i mi . Para A = ( pag q + q pag ) / 2 = i X d / d X i 1 / 2 . Esto da A ( 1 / X z ) = i ( z 1 / 2 ) ( 1 / X z ) . Ir desde A a H se hace usando el operador Δ , lo que lleva a la ecuación formal H ζ 1 / 2 + i mi = mi ζ z . La bc de Dirichlet en el origen obliga a E a satisfacer ζ ( 1 / 2 + i mi ) = 0 .

2)- El primer problema que le veo es que ζ z no parece pertenecer al espacio de Hilbert H = L 2 ( 0 , ) . Usando un método clásico de Hadamard

0 | ζ ( z , X ) | 2 d X = 1 1 2 z ζ ( 2 z 1 )

que converge para z > 1 . Usando la representación integral, se puede obtener una firmula similar, pero no pude demostrar que la integrabilidad cuadrada se mantiene para z = 1 / 2 . Por lo tanto, veo un problema aquí.

3)- El otro problema es la definición de Δ . Dejar S denotan el operador de traducción, definido no rigurosamente por S F ( X ) = F ( X 1 ) . Si se restringe a H , no es unitario, porque se define sólo para X > 1 . Podemos definirlo imponiendo S F ( X ) = 0 para X [ 0 , 1 ] . Entonces es sólo una isometría parcial. Para S F ( X ) = F ( X + 1 ) llevando a S S = I , pero S S = PAG es la proyección sobre L 2 ( 1 , ) . Usando estas notaciones Δ = I S = ( S I ) S . Pero tenemos un problema aquí: la función ζ z se define para X > 1 , y la extensión del intervalo ( 1 , 0 ] se usa explícitamente en 1)-. Entonces no podemos usar S . Pero entonces, ¿cuál es el operador? mi i pag ^ utilizado por los autores?

4)- Si pag ^ es el operador habitual

pag ^ = i d d X

entonces hay un problema con su dominio de definición. En L 2 ( R ) , es autoadjunto como se puede ver usando la transformada de Fourier. pero en H , No lo es. Este es un ejercicio clásico que se encuentra en el libro muy antiguo de Courant-Hilbert. Es decir, uno siempre puede definirlo en el conjunto de L 2 funciones con L 2 derivada, desapareciendo en X = 0 . Entonces es simétrico. Si es así, su adjunto se define en el mismo espacio pero sin desaparecer en X = 0 . No solo el adjunto no es simétrico, sino que su conjunto de valores propios es el semiplano inferior abierto. Esto es porque si F z ( X ) = mi z X , entonces pag ^ F z = i z F z , mientras F z H para z < 0 . El mismo argumento muestra que + i no puede ser un valor propio. Esto significa que los "índices de defectos", es decir, la dimensión de los espacios propios con valores propios ± i , no son iguales. Entonces, el teorema de von Neumann muestra que el operador pag ^ no tiene extensión selfajoint. Si no es autoadjunto, la definición de su exponencial se convierte en un problema, porque el cálculo funcional no está definido en general.

5)- El argumento anterior se puede reformular en términos del operador S . Su adjunto admite muchos valores propios, a saber, los puntos dentro del disco unitario.

En conclusión, el descuido de las definiciones utilizadas pero los autores conducen a un completo desbarajuste. Nada es correcto en este documento.

Mientras los físicos usen álgebra o argumentos algorítmicos, pueden encontrar resultados sobresalientes. Pero cuando se trata de análisis, pueden perder el juicio y graves errores aparecen en la esquina. El análisis no se presta fácilmente a las descripciones algorítmicas. Y aquí es precisamente donde radica el poder de las Matemáticas: al manipular infinitos, las Matemáticas van mucho más allá de la definición de computabilidad de Church-Turing. ¿Y qué es el Análisis sino manipular infinitos, a través de límites, convergencias y cosas por el estilo?

¿Es posible demostrar que ζ X satisface alguna versión más débil de integrabilidad? Tengo en mente la historia a la que se hace referencia en mi comentario anterior a la pregunta, donde se menciona que el enfoque falla debido a que la función propia deseada se encuentra solo en un + 1 ϵ -indexar el espacio de Sobolev en lugar del necesario + 1 -Índice del espacio de Sobolev.
"...al manipular infinitos, las Matemáticas van mucho más allá de la definición de computabilidad de Church-Turing. ¿Y qué es el Análisis sino manipular infinitos, a través de límites, convergencia y cosas por el estilo?" Estimado Jean, no entiendo esta declaración vaga (ese análisis va más allá de la definición de computabilidad de Church-Turing") y, en la medida en que es significativa, parece incorrecta,
Ahora hay una versión completa de esta respuesta en arXiv, arxiv.org/abs/1704.02644 , con suerte se enviará a PRL como un comentario formal.
La expresión del cuadrado norma de ζ z parece ser negativo para R mi ( z ) > 1 . ¿Hay un error tipográfico?
Como señala @SlavaKashcheyevs, hay una preimpresión en arXiv que elabora cuidadosamente esta respuesta y (en mi opinión) brinda una evaluación muy precisa de las debilidades (fatales, en la forma actual).
Vale la pena leer la respuesta de Bender, Brody y Müller a las críticas de Belissard: arxiv.org/abs/1705.06767
¿Algún comentario de tu parte, Jean, a la respuesta de Bender, Brody y Müller?
@jean-v-bellissard: ¿Podría darme la ref. sobre el uso de un método clásico de Hadamard para calcular 0 | ζ ( z , X ) | 2 d X = 1 1 2 z ζ ( 2 z 1 ) ? ¡Gracias!

No tengo todo escrito en este documento.

Pero tenga en cuenta que esta línea de ataque tiene buenas posibilidades de ser demasiado simple: todo esto funciona igual para F ( s ) = α L ( s , x 5 ) + α ¯ L ( s , x 5 ¯ ) = 2 norte = 1 ( α x 5 ( norte ) ) norte s dónde x 5 es el módulo del personaje no real 5 y α C .

F ( s ) tiene el mismo tipo de representación integral y ecuación funcional que ζ ( s ) , por lo que podemos escribir para él el mismo tipo de operador diferencial. Pero el RH obviamente falla por F ( s ) (no tiene un producto de Euler)

Por cierto, su respuesta fue citada por Peter Woit en su blog: math.columbia.edu/~woit/wordpress/?p=9197
Sería interesante si uno pudiera tomar esto F ( s ) y construya el hamiltoniano correspondiente; mi conjetura sería que el resultado seguiría siendo un operador PT-simétrico, pero con pares de valores propios complejos conjugados en lugar de un espectro puramente real.
Esta observación no es relevante, porque la función F no es un estado propio formal de un operador similar a uno simétrico como el generador de dilatación
Dejar C 1 = 2 R mi ( α ) , C 2 = 2 I metro ( α ) , C 2 = 2 I metro ( α ) , C 4 = 2 R mi ( α ) entonces F ( s ) = 5 s metro = 1 4 C metro ζ ( s , metro 5 ) = 1 Γ ( s ) 0 X s 1 1 mi 5 X metro = 1 4 C metro mi metro X d X . El operador correspondiente Δ y H ^ es ...
Es engañoso decir "La RH obviamente falla por F ( s ) ". No hay nada obvio al respecto. Olvidaste decir que los datos numéricos muestran que RH falla: hay muchos ceros dispersos a la derecha de R mi ( s ) = 1 / 2 , y de hecho hay ceros con parte real mayor que 1 aunque F ( s ) tiene una ecuación funcional que relaciona sus valores en s y 1 s . Mientras F ( s ) falla RH, de ninguna manera falla "obviamente". La falta de un producto de Euler no muestra esto (RH no se trata de productos de Euler) y, además, la razón por la que podemos decir que no hay un producto de Euler es, nuevamente, los datos numéricos sobre ceros.
@KCd F ( s ) tiene un cero en s = σ al elegir α = i L ( σ , x 5 ) ¯ . Y F ( s ) no tiene producto de Euler porque los coeficientes de su serie de Dirichlet no son multiplicativos. Finalmente, el teorema de que L ( s , x 5 ) toma todos los valores excepto 0 en ( 1 ϵ , 1 ) significa que el RH falla por cualquier α .
Buen punto sobre la multiplicatividad. Con respecto a tu último punto: L ( 1 , x 5 ) es finito y distinto de cero, L ( s , x 5 ) no toma todos los valores complejos distintos de cero en un intervalo a la izquierda de 1 . Probablemente su intervalo estaba destinado a ser una franja vertical con partes reales en ese intervalo. Todavía diría que esto no es "obvio".
El artículo de Bombieri-Hejhal muestra que bajo una HR adecuada, hay infinitos ceros fuera de línea de la combinación lineal dada en esta respuesta, y analizó su patrón en detalle. Además, se sabe desde Potter-Titchmarsh que los zetas de Epstein tienen muchos ceros fuera de línea, y parecería aplicarse el mismo argumento...

Es un artículo interesante. El principal problema que puedo ver es que su condición límite invoca la función zeta en sí misma, por lo que este enfoque puede estar incorporando la propia RH desde el principio. El enfoque no parece aportar nada profundo sobre los primos ni la teoría de los números.

Habiendo trabajado en este problema durante varios años, sigo siendo escéptico sobre la idea de Hilbert-Polya. Berry-Keating hizo un trabajo muy interesante e importante. Sin embargo, la conexión de la teoría de la matriz aleatoria parece arrojar dudas sobre Hilbert-Polya en lugar de respaldarla: si las estadísticas de los ceros pueden explicarse mediante la teoría de la matriz aleatoria, es difícil imaginar un hamiltoniano simple, no aleatorio, que reproduzca esta teoría . espectro. Pero esto no es un contraargumento riguroso, por supuesto.

Pero el campo del caos cuántico va en contra de tu intuición, ¿no? Hay muchos hamiltonianos no aleatorios cuyo espectro reproduce estadísticas RMT. De hecho, es una propiedad genérica.
Esto es definitivamente cierto, Marcel, y esto ciertamente mantiene viva la idea de Berry-Keating de que debería ser un hamiltoniano caótico. Soy escéptico de que uno pueda simplemente tener suerte y encontrar este hamiltoniano. Su H = xp es probablemente el más cercano a él, si es que existe. Pero se sabe que no funciona. Hay demasiados detalles en los ceros que un hipotético hamiltoniano tendría que acertar.
Según tengo entendido, este caótico hamiltoniano debe tener los números primos en algún lugar de sus órbitas periódicas. Entonces esto de hecho sería muy difícil de encontrar, estoy de acuerdo.
Medir el espacio seguido de medir el momento, o viceversa, produce inevitablemente un resultado "aleatorio" en la mecánica cuántica (HUP). Esto es lo que me fascina de estos operadores que contienen productos. X ^ pag ^ .

El artículo en cuestión está escrito en el nivel físico de rigor, está publicado en una revista de física y no parece afirmar que establece resultados matemáticamente rigurosos. Sin embargo, no estoy convencido por las críticas hasta el momento de que sea imposible convertir la idea principal del artículo en un enunciado matemáticamente bien definido. L 2 problema espectral cuya solución implicaría HR.

Tal vez algo como lo siguiente puede funcionar.

1) Las funciones propias ψ z ( X ) de H ^ de hecho no están en L 2 ( 0 , ) pero parece que uno puede mapear los que corresponden a los ceros no triviales de ζ como sigue. Dejar w ( X ) Sea una función suave positiva en ( 0 , ) que coincide con 1 / X cerca 0 y con 1 / X 3 2 cerca del infinito y dejar W ^ Sea el operador dado por la multiplicación con w . Entonces W ^ ψ z L 2 ( 0 , ) si y solo si z es un cero no trivial de la función zeta de Riemann. En efecto, W ^ ψ z es integrable en 0 si y solo si ψ z ( 0 ) = 0 , es decir, si z es un cero de ζ . Además, las propiedades asintóticas conocidas de la función zeta de Hurwitz (ya mencionada en el artículo de BBM, a saber, el crecimiento sublineal para z en la franja crítica y un crecimiento mucho más rápido cuando z es un cero trivial) implica que W ^ ψ z es integrable al cuadrado en el infinito solo cuando el cero z no es trivial. (El comportamiento de ψ z ( X ) en el infinito también se analiza en el documento de comentarios de Jean Bellissard arxiv.org/abs/1704.02644, utilizando la representación integral de la función zeta de Hurwitz).

2) Ahora considere el operador H ^ W = W ^ H ^ W ^ 1 , de modo que, al menos formalmente, W ^ ψ z son funciones propias de H ^ W por cada cero no trivial z . Para hacer esto riguroso, uno tiene que demostrar que H ^ W es un operador cerrable densamente definido en L 2 ( 0 , ) y eso W ^ ψ z están en el dominio de su clausura. (Eso, por supuesto, incluiría resolver algunos tecnicismos con respecto a la definición adecuada y el dominio de Δ ^ 1 , el inverso del operador diferencia considerado en el documento de BBM, pero no veo por qué esto sería a priori imposible).

3) Suponiendo que se puede hacer lo descrito en 2), se obtiene una incrustación de los ceros no triviales en el espectro discreto de H ^ W . Ahora se sabe que el hamiltoniano de Berry-Keating X ^ pag ^ + pag ^ X ^ es autoadjunto y, por lo tanto, tiene un espectro real, por lo que uno puede intentar usar el hecho de que H ^ W es parecido a X ^ pag ^ + pag ^ X ^ obtener estimaciones sobre el operador resolutivo que impliquen que el espectro de H ^ W está contenida en el espectro de X ^ pag ^ + pag ^ X ^ .

Finalmente, por supuesto que puede ser que H ^ W tiene muchos otros espectros, incluido el complejo, mientras que su espectro discreto es real, por lo que la RH aún puede ser verdadera pero no accesible a través de esta línea de ataque.

Debe escribir lo que está asumiendo (lo que es ψ z , y por qué W ^ ψ z debería estar en L 2 ( 0 , ) sólo cuando ζ ( z ) = 0 )
Estimado usuario1952009, estoy siguiendo la notación utilizada en la pregunta, ψ z ahí se define. Cuando z no es un cero, por definición W ^ ψ z se comporta como 1 / X cerca de 0

En mi respuesta anterior, dejé de lado la cuestión de cómo uno puede dar sentido rigurosamente a los operadores. Δ ^ = 1 mi i pag ^ y Δ ^ 1 = ( 1 mi i pag ^ ) 1 utilizado en el documento de BBM. Me gustaría profundizar un poco en esto.

1) Como es bien conocido (y explicado en la respuesta de Jean Bellissard) el operador de impulso pag ^ no admite extensiones autoadjuntas en L 2 ( 0 , ) , por lo tanto, uno no puede usar (al menos no directamente) el cálculo funcional para dar sentido a Δ ^ . En el periódico BBM Δ ^ se interpreta como un operador de diferencia y Δ ^ 1 como su inversa definida a través de series infinitas (aparentemente, siguiendo a Euler bastante de cerca), pero este enfoque no es satisfactorio por muchas razones.

2) Sería mucho más apropiado para la idea general de cuantización pensar en Δ ^ y Δ ^ 1 como operadores pseudodiferenciales. Dado que esto no se puede hacer directamente, sugiero proceder de la siguiente manera, utilizando las funciones de corte apropiadas para obtener símbolos con un soporte suave y compacto. Dejar k ε ( pag ) ser una familia de funciones en C C ( R ) que es 1 en [ 1 / ε , 1 / ε ] y 0 fuera de este intervalo, de modo que k ε 1 como ε 0 . Dejar Δ ^ ε sea ​​la familia de operadores pseudodiferenciales (de hecho suavizantes) con símbolos ( 1 mi i pag ) k ε ( pag ) . Además, deja x ε ( pag ) ser otra familia en C C ( R ) cual es 0 en [ ε , ε ] así como fuera [ 1 / ε , 1 / ε ] y 1 en el medio, por lo que todavía uno tiene x ε 1 como ε 0 . Dejar Δ ^ ε 1 sea ​​la familia de operadores pseudodiferenciales con símbolos ( 1 mi i pag ) 1 x ε ( pag ) . (Hay mucha investigación sobre operadores pseudodiferenciales en la línea media y sospecho que parte de ella puede ser muy relevante para este contexto particular).

3) Ahora un límite apropiado (digamos, límite de gráfico débil) como ε 0 de estas familias de operadores pueden existir o no, pero incluso si no existe, me parece plausible que se pueda demostrar que las relaciones formales entre Δ ^ , Δ ^ 1 y la función zeta de Hurwitz dada en el artículo de BBM se mantiene "asintóticamente" como ε 0 . Por lo tanto, lo mismo debería ser cierto para la ecuación formal de valor propio/función propia, de modo que si uno define H ^ ε = Δ ^ ε 1 ( X ^ pag ^ + pag ^ X ^ ) Δ ^ ε , uno tendría

límite ε 0 H ^ ε ψ z = i ( 2 z 1 ) ψ z
como, digamos, límite débil de distribuciones.

4) Finalmente, uno puede mirar los operadores H ^ ε , W = W ^ H ^ ε W ^ 1 , dónde W ^ se definió en mi respuesta anterior. La realidad deseada de los valores propios se seguiría entonces de un uniforme (en ε ) obligado por las normas de los operadores resolutivos de H ^ ε , W por cada fijo no real z . Existen condiciones suficientes bien conocidas para la L 2 -acotación de un operador pseudodiferencial en términos de su símbolo.

Puede editar y expandir su respuesta existente, esto encajaría mejor con el formato de preguntas y respuestas de este sitio.
Sí, lo noté, pero pensé que agregar todo esto a la primera respuesta lo haría demasiado largo.

Soy un teórico de números que ha reflexionado sobre la hipótesis de Riemann durante muchos años. Este documento tiene errores elementales y hay contraejemplos simples al enfoque adoptado. No se habría publicado en una revista de matemáticas, y estoy decepcionado (pero no sorprendido) de que se haya publicado en PRL. Bellissard ha enumerado algunos de los errores en el análisis (ver arXiv:1704.02644) y las locuras de la teoría de números serían claras para cualquier estudiante de posgrado en el campo.

Es interesante desde el punto de vista sociológico que tantos físicos de renombre estén blogueando o tuiteando sobre esto cuando simplemente podrían consultar a expertos en sus universidades. Para esos expertos, esto es una broma.

Un día oscuro para la ciencia.

Bienvenido a Matemáticas Stack Exchange, ¿podría mostrar los "errores elementales" que menciona? de lo contrario, su respuesta es algo infundada
Les he dado una referencia al artículo de Bellissard, que enumera algunos errores. Ya se ha discutido un contraejemplo anteriormente: si toma combinaciones lineales de funciones L que satisfacen la misma ecuación funcional, entonces el enfoque del documento se aplica mutatis mutandis, pero tales combinaciones no satisfacen la RH. (A pesar del hecho de que si cada función L satisface RH, también lo hará el 100% de los ceros de la combinación lineal).
entonces sugiero vincular el documento, porque de lo contrario su respuesta aún no tiene fundamento
ver arXiv:1704.02644
No veo qué aporta esta respuesta. El único punto sustantivo es el artículo de Bellisard, y él ya tiene una respuesta aquí. Así que no veo por qué esta es una respuesta en lugar de un comentario.
@ Semiclassical La mayoría de las respuestas a esta pregunta son de personas nuevas en SE, no están acostumbradas a comentar / responder a la distinción.

Algunas de las principales discrepancias con respecto a la conjetura de Bender-Brody-Müller (BBM) se relacionan con el dominio del operador y la convergencia del espectro propio. Al considerar el problema como un operador de Schrödinger, se puede obtener un espectro propio convergente. Estas soluciones se analizan aquí y aquí .

¿Significa esto que RH lo resuelve Moxley? ¿Consiguió el millón de dólares?
Parece que el RH lo resuelve Moxley. Para obtener el millón de dólares, se necesitaría una nominación o, para empezar, una medalla Fields. imuweb.mathunion.org/general/prizes/fields/detalles
Verifique esto , varios resultados aquí, por ejemplo, en lugar de probar Hilbert-Polyá (una prueba de existencia), ¡obtiene una expresión explícita! Decenas de líneas de cálculos triviales en lugar de cualquier idea nueva. (Quiero decir, ya sabes, no hay señal alguna de que pag ^ X ^ + X ^ pag ^ es en realidad hermitiano).
Es interesante notar que se ha proporcionado una prueba: hal.archives-ouvertes.fr/hal-01804653/document

Solo un pensamiento que me gustaría desarrollar aquí, inspirado en la respuesta de @AndréLeClair (es demasiado largo para un comentario).

La pregunta básica es: ¿Cuál es la conexión entre la teoría de matrices aleatorias (donde se recuperan algunas estadísticas básicas de los ceros de la función zeta) y la (apenas) PAG T operador simétrico (en lugar de auto-adjunto) de Bender-Brody-Müller (BBM)?

Parece que, idealmente , la teoría de la matriz aleatoria sería exactamente la formulación de la matriz de Heisenberg de la formulación del "tipo Schrödinger" de BBM.

¿Es posible que eso sea cierto teniendo en cuenta que el operador de BBM se ve tan norte o norte - r a norte d o metro ?

Tal vez: el principio de incertidumbre de Heisenberg nos dice que ningún objeto cuántico puede tener un momento y una posición definidos en el mismo estado. Medir el impulso y la posición uno después del otro conduce inevitablemente a un resultado para el cual solo se pueden dar distribuciones de probabilidad en lugar de valores definidos.

¿Tendría sentido interpretar los operadores Berry-Keating (o derivados) que contienen productos pag ^ X ^ tal que uno forzaría al sistema a estados propios con un producto localizado (definido) de cantidad de movimiento y posición? Uno entonces tendría que leer estos términos como pag ^ ( X ^ ( ψ ) ) Lo que lleva a una especie de estado paradójico, que ya se refleja en que estos operadores no son autoadjuntos.

(No estoy seguro si este argumento ya ha sido considerado por BBM).

Podemos probar la hipótesis de Riemann usando el operador GRAMO ^ + GRAMO ^ , definido como

GRAMO ^ = pag ^ Δ ^ X ^ Δ ^ +

GRAMO ^ = Δ ^ X ^ Δ ^ + pag ^
Δ ^ + es el operador de diferencia directa y Δ ^ es el operador de diferencia hacia atrás.

Si r y s son ceros de la zeta de Riemann con partes reales mayores que 1 2 , podemos demostrar que pag ^ es auto adjunto y eso

φ r | GRAMO ^ | φ s = φ r | GRAMO ^ | φ s
GRAMO ^ + GRAMO ^ está relacionado con el hamiltoniano BBM H ^ = Δ ^ + 1 ( X ^ pag ^ + pag ^ X ^ ) Δ ^ + por
GRAMO ^ + GRAMO ^ = Δ ^ Δ ^ + H ^
Porque φ s son funciones propias de H ^ con valores propios i ( 1 2 s ) , el valor esperado de GRAMO ^ + GRAMO ^ es
φ s | GRAMO ^ + GRAMO ^ | φ s = i ( 1 2 s ) φ s | Δ ^ Δ ^ + | φ s
Y φ s | Δ ^ Δ ^ + | φ s converge para R mi ( s ) > 1 2 .

Esta ecuación es válida si s es un cero de Riemann zeta con parte real mayor que 1 2 . Entonces, si asumimos que existe un cero con parte real mayor que 1 2 , entonces el valor propio i ( 1 2 s ) tiene que ser real, por lo que la parte real de s tiene que ser 1 2 . Esta contradicción significa que no puede haber ceros con parte real mayor que 1 2 . ¡La hipótesis de Riemann es cierta!

Editar: la última ecuación seguiría siendo válida para s no siendo un cero de Riemann zeta. Pero entonces GRAMO ^ = Δ ^ X ^ Δ ^ + pag ^ no sería el hermitiano de GRAMO ^ = pag ^ Δ ^ X ^ Δ ^ + y luego φ s | GRAMO ^ + GRAMO ^ | φ s probablemente no sería real. Puede leer en mi artículo por qué la ermitancia está ahí para la condición de contorno φ s ( 1 ) = ζ ( s ) = 0 con R mi ( s ) > 1 2 .

Bueno, ¿qué pasa con las partes reales menos de 1 2 ? No puedo seguir esto lo suficiente como para decir si algo más está mal con esto. Además, hacerlo en unos pocos párrafos probablemente signifique que algo anda muy mal.
De la ecuación funcional ζ ( s ) = 2 s π s 1 pecado ( π s 2 ) Γ ( 1 s ) ζ ( 1 s ) , si s es un cero de la función zeta de Riemann con parte real entre 0 y 1, entonces 1 s también sería un cero.
Bueno, ¿qué pasa con esto, Γ ( 1 s ) ? ¿Puedes probar que nunca será un cero? Estoy bastante seguro de que algo más está mal, pero de nuevo, no tengo suficiente experiencia para saberlo. Es corto (lo cual es parte del problema), por lo que no será demasiado difícil para las personas repasarlo.
De la fórmula de reflexión de Euler Γ ( s ) Γ ( 1 s ) = π / s i norte ( π s ) sabemos que la función Gamma no tiene ceros en el plano complejo con parte imaginaria distinta de cero.
Cuando dices: "Y φ s | Δ ^ Δ ^ + | φ s converge para R mi ( s ) > 1 2 ". ¿Cómo sabes eso? ¿Se ha probado antes? ¿Cuáles son tus fuentes?
Gracias por su pregunta. φ s | Δ ^ Δ ^ + | φ s = φ s | Δ ^ + Δ ^ + | φ s = Δ ^ + φ s | Δ ^ + φ s = 1 X 1 X s | 1 X 1 X s = 1 d X   X 2 R mi ( s ) que converge si s tiene una parte real mayor que 1 2 .
No, quise decir que debes probarlo matemáticamente o dar una fuente que lo diga. No puedes decir cosas así.
¿Sobre qué paso de la demostración le gustaría que profundizara?
Cuando pruebes la convergencia.
¿Quieres decir esto? 1 d X   X 2 R mi ( s ) = 1 1 2 R mi ( s ) [ X 1 2 R mi ( s ) ] 1 = 1 2 R mi ( s ) 1
Sí. Tienes que demostrarlo. No puedes esperar que la gente simplemente crea que lo hiciste sin alguna prueba.
Te mostré todos los pasos del cálculo de la convergencia. ¿Qué más quieres que pruebe? ¿O en qué paso quieres que te explique?
Echa un vistazo a esta prueba de la hipótesis de Riemann linkedin.com/pulse/finding-hamiltonian-hung-tran
Por favor, mira mi comentario de tu pregunta.
¿Cuál es el estado de su prueba Hung Trang?
@PtF hay un problema con la discontinuidad en x = 2. Traté de solucionarlo.

La situación actual es la siguiente. El proyecto zetagrid ya no existe. La función Z tiene un polo en Re (z) = 1, pero es analítica para Re (z) > 1 y divergente para Re (z) < 1. Riemann extendió analíticamente la función para que pudiera ser analítica en la región 0 < Re (z) < 1, que es lo que se conoce como franja crítica o intervalo de investigación. Donde tenemos que para Re (z) = 1/2 hay infinitos ceros (Hardy, 1914) y se supone (RH) que todos los ceros de la Z están ahí. Sin embargo, en noviembre de 2017, este servidor mostró que existen infinitos ceros de la Z en el intervalo crítico distribuidos en diferentes familias de líneas, ¡con lo cual la Hipótesis de Riemann es falsa! Además, se confirmó la existencia de los ceros por reflexión predichos por la ecuación funcional de Riemann y según los cuales si en el intervalo 0 <Ingrese la descripción de la imagen aquí 0.44 + 1977.19-> 0.56 + 1977.19 0.45 + 1329.09-> 0.55 + 1329.09 0.45 + 1415.59-> 0.55 + 1415.59 0.46 + 25.0-> 0.54 + 25.0 0.46 + 37.6-> 0.54 + 37.6 0.47 + 14.09-> 0.53 + 14,09 0,47 + 21,0-> 0,53 + 21,0 0,48 + 14,09-> 0,52 + 14,09 0,48 + 21,0-> 0,52 + 21,0 0,49 + 14,09-> 0,51 + 14,09 0,49 + 21,0-> 0,51 +

todos ellos son ceros de la Z (por reflexión), que tampoco están en la línea Re (z) = 1/2, como es fácil de ver y verificar usando mis expresiones para la función Z. Te dejo esa tarea a ti.

Uno, no usaste Latex y no mostraste ninguna fuente. Tienes que hacer estos dos para que te tomen en serio.